討論串[問題] 問一個關於量子力學eigenstate的問題
共 2 篇文章
首頁
上一頁
1
下一頁
尾頁

推噓7(7推 0噓 34→)留言41則,0人參與, 5年前最新作者pennyleo (那些物理高手好厲害)時間13年前 (2010/09/26 10:16), 編輯資訊
0
0
0
內容預覽:
在測量一物理狀態. 因為測出的結果必是實數. 所以我們"要求"這個operator要為hermitian. 但我的問題是. 在矩陣運算裡. 如果一矩陣不是hermitian 算出的所有eigenvalue仍有可能全是實數. 這是否表一物理測量值有可能對應到不是hermitian的算符??. 高手可否

推噓1(1推 0噓 8→)留言9則,0人參與, 最新作者pennyleo (落日黃花)時間13年前 (2011/03/14 16:58), 編輯資訊
0
0
0
內容預覽:
有件事想請各位幫忙. 我於去年的9/26日問了有關hermitian算符的問題. 很感激當時有些人提出了一些解釋. 我想用這個問題 當作我期中報告的主要問題之一. 但對這個問題 自己仍然並不是非常清楚. 我想問. 版上是否有人可以推薦我關於這個問題的參考書籍. 我確定shankar和sakurai上
首頁
上一頁
1
下一頁
尾頁